2
$\begingroup$

Let $a_{1},\dots,a_{n}$ be positive natural numbers ($n>2$) such that $a_{i}\neq a_{j}$ if $i\neq j$. I want to prove that $$ \left\lvert \left\{ p \text{ prime} \; : \; p \mid \sum_{i=1}^n a_{i}^{k} \text{ for some } k\in\mathbb{N} \right\} \right\rvert = \infty . $$ I considered $p > a_{i},\,\forall i=1,\dots,r$ and $k=\frac{p-1}{2}$ so $$ \sum_{i=1}^n a_{i}^{k} \equiv \sum_{i=1}^n \left(\frac{a_{i}}{p}\right) \bmod p $$ where $\left(\frac{*}{*}\right)$ is the Legendre symbol. But now I'm stuck because nothing guarantees that $$\sum_{i=1}^n \left(\frac{a_{i}}{p}\right) \equiv 0 \bmod p . $$ Thanks.

$\endgroup$
5
  • 2
    $\begingroup$ This follows from the finiteness of solutions to the $S$-unit equation. See for example the main theorem in math.uwaterloo.ca/pure-mathematics/sites/ca.pure-mathematics/… . Maybe there's an easier proof ... $\endgroup$
    – Lucia
    Dec 20, 2014 at 10:21
  • $\begingroup$ The phrase " narrowness implies uniformity" comes to mind. This may be a result of McKenzie. I will check on it later. $\endgroup$ Dec 20, 2014 at 10:36
  • $\begingroup$ @lucia Sorry but I can't see how this can help me... Can you give me an hint? $\endgroup$
    – peppo
    Dec 20, 2014 at 10:45
  • $\begingroup$ Assume that only finitely many primes occur. Let $S$ be the set consisting of those primes together with the prime divisors of the $a_i$. Then $a_i^k$ for all $i$ and $b = a_1^k + \ldots + a_n^k$ are $S$-units, and we have the relation $a_1^k + \ldots + a_n^k - b = 0$. The main result on $S$-unit equations says that there are only finitely many solutions (up to scaling by $S$-units) such that no nontrivial sub-sum vanishes. This condition follows in your case from $a_i > 0$. $\endgroup$ Dec 20, 2014 at 15:35
  • $\begingroup$ A stronger result for the case $k=3$ seems to be called "Reutter's theorem". A question on MSE asking a reference for it (and a link to AoPS with an elementary proof) can be found here math.stackexchange.com/questions/1475267/… $\endgroup$ Oct 11, 2015 at 20:15

1 Answer 1

4
$\begingroup$

Without loss of generality, we can assume that there is no prime dividing all $a_i$. For each prime $p$, denote by $n_p$ the number of those $a_i$ not divisible by $p$, and let $v_p$ be the power to which $p$ divides $n_p$. (Notice that $n_p>0$ by the assumption that no prime divides all $a_i$; hence, $v_p$ are well-defined.) By Euler's theorem from the elementary number theory, we have $$ a_i^{p^{v_p}(p-1)}\equiv 1\pmod {p^{v_p+1}} $$ for each $a_i$ not divisible by $p$, implying $$ \sum_{i=1}^n a_i^k\equiv n_p\not\equiv 0\pmod {p^{v_p+1}} $$ whenever $k$ is a multiple of $p^{v_p}(p-1)$. It follows that for every finite set $P$ of primes, if $k$ is divisible by the product $\prod_{p\in P} p^{v_p}(p-1)$, then each prime $p\in P$ divides the sum $\sum_{i=1}^n a_i^k$ to the power not exceeding $v_p\le\log_p n$. Taking $k$ sufficiently large, we see that this sum is just too large to have all its prime divisors in $P$. In other words, for every finite set $P$ of primes, there exists $k$ such that the sum in question has a prime divisor outside of $P$. This shows that the set of all prime divisors of all these sums is infinite.

Incidentally, this argument does not use the fact that all $a_i$ are pairwise distinct; it suffices that not all of them are equal to each other (which is easily seen to be a necessary condition, too).

$\endgroup$
0

Your Answer

By clicking “Post Your Answer”, you agree to our terms of service and acknowledge you have read our privacy policy.

Not the answer you're looking for? Browse other questions tagged or ask your own question.